初等数论练习题答案

初等数论练习题答案
初等数论练习题答案

初等数论练习题答案

信阳职业技术学院

2010年12月

初等数论练习题一

一、填空题

1、d(2420)=12; ?(2420)=_880_

2、设a,n 是大于1的整数,若a n -1是质数,则a=_2.

3、模9的绝对最小完全剩余系是_{-4,-3,-2,-1,0,1,2,3,4}.

4、同余方程9x+12≡0(mod 37)的解是x ≡11(mod 37)。

5、不定方程18x-23y=100的通解是x=900+23t ,y=700+18t t ∈Z 。.

6、分母是正整数m 的既约真分数的个数为_?(m )_。

7、18100被172除的余数是_256。

8、??? ??10365 =-1。 9、若p 是素数,则同余方程x p - 1 ≡1(mod p )的解数为 p-1 。

二、计算题

1、解同余方程:3x 2+11x -20 ≡ 0 (mod 105)。

解:因105 = 3?5?7,

同余方程3x 2

+11x -20 ≡ 0 (mod 3)的解为x ≡ 1 (mod 3),

同余方程3x 2+11x -38 ≡ 0 (mod 5)的解为x ≡ 0,3 (mod 5),

同余方程3x 2+11x -20 ≡ 0 (mod 7)的解为x ≡ 2,6 (mod 7),

故原同余方程有4解。

作同余方程组:x ≡ b 1 (mod 3),x ≡ b 2 (mod 5),x ≡ b 3 (mod 7),

其中b 1 = 1,b 2 = 0,3,b 3 = 2,6,

由孙子定理得原同余方程的解为x ≡ 13,55,58,100 (mod 105)。

2、判断同余方程x 2≡42(mod 107)是否有解?

11074217

271071107713231071107311072107

710731072107732107422110721721107213)(=∴-=-=-==-=-=-==??≡-?--?-)()()()(),()()()(),()())()(()(解: 故同余方程x 2≡42(mod 107)有解。

3、求(127156+34)28除以111的最小非负余数。

解:易知1271≡50(mod 111)。

由502 ≡58(mod 111), 503 ≡58×50≡14(mod 111),509≡143≡80(mod 111)知5028 ≡(509)3×50≡803×50≡803×50≡68×50≡70(mod 111)

从而5056 ≡16(mod 111)。

故(127156+34)28≡(16+34)28 ≡5028≡70(mod 111)

三、证明题

1、已知p 是质数,(a,p )=1,证明:

(1)当a 为奇数时,a p-1+(p-1)a

≡0 (mod p);

(2)当a 为偶数时,a p-1-(p-1)a ≡0 (mod p)。

证明:由欧拉定理知a p-1≡1 (mod p)及(p-1)a ≡-1 (mod p)立得(1)和(2)成立。

2、设a 为正奇数,n 为正整数,试证n 2a ≡1(mod 2n+2)。 (1)

证明 设a = 2m + 1,当n = 1时,有

a 2 = (2m + 1)2 = 4m (m + 1) + 1 ≡ 1 (mod 23),即原式成立。

设原式对于n = k 成立,则有

k a 2≡ 1 (mod 2k + 2) ?k a 2= 1 + q 2k + 2, 其中q ∈Z ,所以 12+k a = (1 + q 2k + 2)2 = 1 + q '2k + 3 ≡ 1 (mod 2k + 3),

其中q '是某个整数。这说明式(1)当n = k + 1也成立。

由归纳法知原式对所有正整数n 成立。

3、设p 是一个素数,且1?k ?p-1。证明:k

p 1C - ≡ (-1 )k (mod p )。

证明:设A=!

)()2(1C 1k k p p p k

p ---=- )( 得: k!·A =(p-1)(p-2)…(p-k )≡(-1)(-2)…(-k )(mod p )

又(k!,p )=1,故A = k p 1

C - ≡ (-1 )k (mod p ) 4、设p 是不等于3和7的奇质数,证明:p 6≡1(mod 84)。

说明:因为84=4×3×7,所以,只需证明:

p 6≡1(mod 4) p 6≡1(mod3) p 6≡1(mod 7) 同时成立即可。

证明:因为84=4×3×7及p 是不等于3和7的奇质数,所以

(p ,4)=1,(p ,3)=1,(p ,7)=1。

由欧拉定理知:p ?(4)≡p 2≡1(mod 4),从而 p 6≡1(mod 4)。

同理可证:p 6≡1(mod3) p 6≡1(mod 7)。 故有p 6≡1(mod 84)。

注:设p 是不等于3和7的奇质数,证明:p 6≡1(mod 168)。(见赵继源p86)

初等数论练习题二

一、填空题

1、d(1000)=_16_;σ(1000)=_2340_.

2、2010!的标准分解式中,质数11的次数是199__.

3、费尔马(Fermat)数是指Fn=n

22+1,这种数中最小的合数Fn 中的n=5。

4、同余方程13x ≡5(mod 31)的解是x ≡29(mod 31)___

5、分母不大于m 的既约真分数的个数为?(2)+ ?(3)+…+ ?(m )。

6、设7∣(80n -1),则最小的正整数n=_6__.

7、使41x+15y=C 无非负整数解的最大正整数C=__559__.

8、??? ??10146=_1__. 9、若p 是质数,n ∣p - 1,则同余方程x n ≡ 1 (mod p ) 的解数为n .

二、计算题

1、试求200420032002被19除所得的余数。

解:由2002≡7 (mod 19) 20022≡11(mod 19) 20023≡1 (mod 19)

又由20032004≡22004≡(22)1002≡1 (mod 3)可得:

200420032002≡20023n+1≡(20023)n ×2002≡7(mod 19)

2、解同余方程3x 14 + 4x 10 + 6x - 18 ≡ 0 (mod 5)。

解:由Fermat 定理,x 5 ≡ x (mod 5),因此,原同余方程等价于2x 2 + x - 3 ≡ 0 (mod 5) 将x ≡ 0,±1,±2 (mod 5)分别代入上式进行验证,可知这个同余方程解是x ≡ 1 (mod 5)。

3、已知a=5,m=21,求使a x ≡ 1 (mod m)成立的最小自然数x 。

解:因为(5,21)=1,所以有欧拉定理知5?(21)≡1(mod 21)。

又由于?(21)=12,所以x |12,而12的所有正因数为1,2,3,4,6,12。

于是x 应为其中使 5 x ≡ 1 (mod 12)成立的最小数,经计算知:x=6。

三、证明题

1、试证13|(54m +46n +2000)。(提示:可取模13进行计算性证明)

证明:54m +46n +2000 ≡ 252m +642n +2000 ≡(-1)2m +(-1)2n +2000 ≡ 2002≡ 0(mod 13)。

2、证明Wilson 定理的逆定理:若n > 1,并且(n - 1)! ≡ -1 (mod n ),则n 是素数。 证明:假设n 是合数,即n = n 1n 2,1 < n 1 < n ,由题设易知(n - 1)! ≡ -1 (mod n 1),得

0 ≡ -1 (mod n 1),矛盾。故n 是素数。

3、证明:设p s 表示全部由1组成的s 位十进制数,若p s 是素数,则s 也是一个素数。 证明:假设s 是合数,即s=ab ,1

M p a b a s s

s ?-=-=-==911091)10(9110111 ,其中M >1是正整数。 由p a >1也是正整数知p s 是合数,这与题设矛盾。故s 也是一个素数。

4、证明:若2p + 1是奇素数,则 (p !)2 + (-1)p ≡ 0 (mod 2p + 1)。

证明:由威尔逊定理知 -1 ≡ (2p )! = p !(p + 1) (2p ) ≡ (-1)p (p !)2(mod 2p + 1),

由此得(p !)2 + (-1)p ≡ 0 (mod 2p + 1)。

5、设p 是大于5的质数,证明:p 4≡1(mod 240)。(提示:可由欧拉定理证明)

证明:因为240=23×3×5,所以只需证:p 4≡1(mod 8),p 4≡1(mod 3),p 4≡1(mod 5)即

可。事实上,由?(8)=4,?(3)=2,?(5)=4以及欧拉定理立得结论。

初等数论练习题三

一、单项选择题

1、若n >1,?(n )=n-1是n 为质数的( C )条件。

A.必要但非充分条件

B.充分但非必要条件

C.充要条件

D.既非充分又非必要条件

2、设n 是正整数,以下各组a ,b 使a

b 为既约分数的一组数是( D )。

A.a=n+1,b=2n-1

B.a=2n-1,b=5n+2

C.a=n+1,b=3n+1

D.a=3n+1,b=5n+2

3、使方程6x+5y=C 无非负整数解的最大整数C 是( A )。

A.19

B.24

C.25

D.30 4、不是同余方程28x ≡21(mod 35)的解为( D )。

A.x ≡2(mod 35)

B. x ≡7(mod 35)

C. x ≡17(mod 35)

D. x ≡29(mod 35)

5、设a 是整数,(1)a ≡0(mod9) (2)a ≡2010(mod9)

(3)a 的十进位表示的各位数字之和可被9整除

(4)划去a 的十进位表示中所有的数字9,所得的新数被9整除

以上各条件中,成为9|a 的充要条件的共有( C )。

A.1个

B.2个

C.3个

D.4个

二、填空题

1、σ(2010)=_4896____;?(2010)=528。

2、数20100C 的标准分解式中,质因数7的指数是_3。

3、每个数都有一个最小质因数。所有不大于10000的合数的最小质因数中,最大者是97。

4、同余方程24x ≡6(mod34)的解是x 1≡13(mod34) x 2≡30(mod34)_。

5、整数n>1,且(n-1)!+1≡0(mod n),则n 为素数。

6、3103被11除所得余数是_5_。

7、??

? ??9760=_-1_。 三、计算题

1、判定 (ⅰ) 2x 3 - x 2 + 3x - 1 ≡ 0 (mod 5)是否有三个解;

(ⅱ) x 6 + 2x 5 - 4x 2 + 3 ≡ 0 (mod 5)是否有六个解?

解:(ⅰ) 2x 3 - x 2 + 3x - 1 ≡ 0 (mod 5)等价于x 3 - 3x 2 + 4x - 3 ≡ 0 (mod 5),又x 5 - x = (x 3 - 3x 2 + 4x - 3)(x 2 + 3x + 5) + (6x 2 - 12x + 15),其中r (x ) = 6x 2 - 12x + 15的系数不都是5的倍数,故原方程没有三个解。

(ⅱ) 因为这是对模5的同余方程,故原方程不可能有六个解。

2、设n 是正整数,求1223212C ,,C ,C -n n

n n 的最大公约数。 解:设12122321212232122C C C )C ,,C ,(C ---=+++=n n n n n n n n n d ,由知d ∣22n -1,

设2k |n 且2k+1|/

n ,即2k +1||n , 则由2k +1||1122112C 2C 2C |--+=i n i n k n i

n 及,i = 3, 5, , 2n - 1 得d = 2k + 1。 3、已知a=18,m=77,求使a x ≡ 1 (mod m)成立的最小自然数x 。

解:因为(18,77)=1,所以有欧拉定理知18?(77)≡1(mod 77)。

又由于?(77)=60,所以x |60,而60的所有正因数为1,2,3,4,5,6,10,12,15,20,30, 60。

于是x 应为其中使18x ≡ 1 (mod 77)成立的最小数,经计算知:x=30。

四、证明题

1、若质数p ?5,且2p+1是质数,证明:4p+1必是合数。

证明:因为质数p ?5,所以(3,p )=1,可设p=3k+1或p=3k+2。

当p=3k+1时,2p+1=6k+3是合数,与题设矛盾,从而p=3k+2,

此时2p+1是形如6k+5的质数,而4p+1=12k+9=3(4k+3)是合数。

注:也可设p=6k+r ,r=0,1,2,3,4,5。再分类讨论。

2、设p 、q 是两个大于3的质数,证明:p 2≡q 2(mod 24)。

证明:因为24=3×8,(3,8)=1,所以只需证明:

p 2≡q 2(mod 3) p 2≡q 2(mod 8)同时成立。

事实上, 由于(p ,3)=1,(q ,3)=1,所以p 2≡1(mod 3) , q 2≡1(mod 3), 于是p 2≡q 2(mod 3),由于p ,q 都是奇数,所以p 2≡1(mod 8) , q 2≡1(mod 8),

于是p 2≡q 2(mod 8)。故p 2≡q 2(mod 24)。

3、若x,y ∈R + ,

(1)证明:[xy ]?[x][y];

(2)试讨论{xy}与{x}{y}的大小关系。

注:我们知道,[x + y ] ?[x ]+ [y ],{x+y }?{x }+{y }。此题把加法换成乘法又如何呢? 证明:(1)设x=[x]+α,0?α<1,y=[y]+β,0?β<1。于是

xy=[x][y]+β[x]+α[y]+αβ

所以[xy ]= [x][y]+ [β[x]+α[y]+αβ] ?[x][y]。

(2){xy}与{x}{y}之间等于、大于、小于三种关系都有可能出现。

当x=y=

21时,{xy}={x}{y}=4

1; 当x=23, y=21时,{xy}=43,{x}{y}=4

1,此时{xy}>{x}{y}; 当x=-21,y=-31时,{xy}=61,{x}{y}=3

1,此时{xy}<{x}{y}。 4、证明:存在一个有理数d c ,其中d < 100,能使 ][d c k =][10073k 对于k=1,2,….,99均成立。

证明:由(73,100)=1以及裴蜀恒等式可知:存在整数c ,d ,使得

73d-100c=1 从而

k 10073-d kc =d c d k 100)10073(-=d k 100,由k< 100可知:

0<

k 10073-d kc <d 1 设][d c k =n ,则d

kc <n+1=d d n 1+,于是 k 10073<d

kc 1+?d d n 1+=n+1, 故 ][10073k = n =][d c k 。

初等数论练习题四

一、单项选择题

1、若F n =12n 2+是合数,则最小的n 是( D )。

A. 2

B. 3

C. 4

D. 5

2、记号b a ‖a 表示b a |a,但b a+1|/a. 以下各式中错误的一个是( B )。

A. 218‖20!

B. 105‖50!

C. 119‖100!

D. 1316‖200!

3、对于任意整数n ,最大公因数(2n+1,6n-1)的所有可能值是( A )。

A. 1

B. 4

C. 1或2

D. 1,2或4

4、设a 是整数,下面同余式有可能成立的是( C )。

A. a 2≡2 (mod 4)

B. a 2≡5 (mod 7)

C. a 2≡5 (mod 11)

D. a 2≡6 (mod 13)

5、如果a ≡b(mod m),c 是任意整数,则下列错误的是( A )

A .ac ≡bc(mod mc)

B .m|a-b

C .(a,m)=(b,m)

D .a=b+mt,t ∈Z 二、填空题

1、d(10010)=_32__,φ(10010)=_2880__。

2、对于任意一个自然数n ,为使自N 起的n 个相继自然数都是合数,可取N=(n+1)!

3、为使3n-1与5n+7的最大公因数达到最大可能值,整数n 应满足条件n=26k+9,k ∈Z 。

4、在5的倍数中,选择尽可能小的正整数来构成模12的一个简化系,则这组数是{5,25,35,55}

5、同余方程26x+1≡33 (mod 74)的解是x 1≡24(mod74) x 2≡61(mod74)_。

6、不定方程5x+9y=86的正整数解是_x=1,y=9或x=10,y=4。

7、???

??8954=_-1_。

三、计算题

1、设n 的十进制表示是z xy 4513,若792∣n ,求x ,y ,z 。

解:因为792 = 8?9?11,故792∣n ? 8∣n ,9∣n 及11∣n 。

我们有8∣n ? 8∣z 45

? z = 6,以及 9∣n ? 9∣1 + 3 + x + y + 4 + 5 + z = 19 + x + y ? 9∣x + y + 1, (1)

11∣n ? 11∣z - 5 + 4 - y + x - 3 + 1 = 3 - y + x ? 11∣3 - y + x 。 (2)

由于0 ≤ x , y ≤ 9,所以由式(1)与式(2)分别得出

x + y + 1 = 9或18,

3 - y + x = 0或11。

这样得到四个方程组:???=+-=++b

x y a y x 31 其中a 取值9或18,b 取值0或11。在0 ≤ x , y ≤ 9的条件下解这四个方程组, 得到:x = 8,y = 0,z = 6。

2、求3406的末二位数。

解:∵ (3,100)=1,∴3φ(100)≡1(mod 100),而φ(100)= φ(22·52)=40,

∴ 340≡1(mod 100) ∴ 3406=(340)10·36≡(32)2·32≡-19×9≡-171≡29(mod 100) ∴ 末二位数为29。

3、求(214928+40)35被73除所得余数。

解:(214928+40)35≡(3228+40)35≡[(32×32)14+40]35 ≡(102414+40)35 ≡(214+40)35 ≡(210

×24+40)35 ≡(25+40)35 ≡7235 ≡-1≡72(mod 73)

四、证明题

1、设a 1, a 2, , a m 是模m 的完全剩余系,证明:

(1)当m 为奇数时,a 1+ a 2+ + a m ≡0(mod m );

(2)当m 为偶数时,a 1+ a 2+ + a m ≡2

m (mod m )。 证明:因为{1, 2, , m }与{a 1, a 2, , a m }都是模m 的完全剩余系,所以

∑∑==+=

≡m i m i i m m i a 112

)1((mod m )。

(1)当m 为奇数时,Z m ∈+21由即得:2)1(+m m m ,故02)1(1

≡+=∑=m i i m m a (mod m )。 (2)当m 为偶数时,由(m,m+1)=1即得:2

2)1(1m m m a m

i i ≡+=

∑=(mod m )。 2、证明:若m >2,a 1, a 2, , a ?(m)是模m 的任一简化剩余系,则 ).(mod 01m a

m i i ∑=≡)

(?

证明:若a 1, a 2, , a ?(m)是模m 的一个简化剩余系,则m-a 1, m-a 2, , m-a ?(m) 也是模m

的一个简化剩余系,于是: ).(mod )()(11m a m a m i i

m i i ∑∑==-≡??)

( 从而:).)(mod (21m m m a m i i ??∑=≡)(

又由于m >2,?(m )是偶数。故:).(mod 021m m m a m i i ≡≡∑=)()

(??

3、设m > 0是偶数,{a 1, a 2, , a m }与{b 1, b 2, , b m }都是模m 的完全剩余系,证明:{a 1 + b 1,

a 2 +

b 2, , a m + b m }不是模m 的完全剩余系。

证明:因为{1, 2, , m }与{a 1, a 2, , a m }都是模m 的完全剩余系,所以

∑∑==≡+=

≡m i m i i m m m i a 112

2)1((mod m )。 (1) 同理 ∑=≡

m i i m b 12

(mod m )。 (2) 如果{a 1 + b 1, a 2 + b 2, , a m + b m }是模m 的完全剩余系,那么也有

∑=≡+m

i i i m b a 1

2)((mod m )。 联合上式与式(1)和式(2),得到 02

22m m m ≡+≡(mod m ), 这是不可能的,所以{a 1 + b 1, a 2 + b 2, , a m + b m }不能是模m 的完全剩余系。

4、证明:(1)2730∣x 13-x ; (2)24∣x(x+2)(25x 2-1);

(3)504∣x 9-x 3; (4)设质数p >3,证明:6p ∣x p -x 。

证明:(1)因为2730=2×3×5×7×13,2,3,5,7,13两两互质,所以: 由x 13-x=x (x 12-1)≡0 (mod 2)知:2∣x 13-x ;13∣x 13-x ;

由x 13-x=x (x 12-1)=x(x 2-1)(x 2+1)(x 8+x 4+1)≡0 (mod 3)知:3∣x 13-x ;

由x 13-x=x (x 12-1)=x(x 4-1)(x 8+x 4+1)≡0 (mod 5)知:5∣x 13-x ;

初等数论练习题及答案

初等数论练习题一 一、填空题 1、τ(2420)=27;?(2420)=_880_ 2、设a ,n 是大于1的整数,若a n -1是质数,则a=_2. 3、模9的绝对最小完全剩余系是_{-4,-3,-2,-1,0,1,2,3,4}. 4、同余方程9x+12≡0(mod 37)的解是x ≡11(mod 37)。 5、不定方程18x-23y=100的通解是x=900+23t ,y=700+18t t ∈Z 。. 6、分母是正整数m 的既约真分数的个数为_?(m )_。 7 8、??? ??10365 =-1。 9、若p 是素数,则同余方程x p - 1 ≡1(mod p )的解数为二、计算题 1、解同余方程:3x 2+11x -20≡0 (mod 105)。 解:因105 = 3?5?7, 同余方程3x 2+11x -20≡0 (mod 3)的解为x ≡1 (mod 3), 同余方程3x 2+11x -38 ≡0 (mod 5)的解为x ≡0,3 (mod 5), 同余方程3x 2+11x -20≡0 (mod 7)的解为x ≡2,6 (mod 7), 故原同余方程有4解。 作同余方程组:x ≡b 1 (mod 3),x ≡b 2 (mod 5),x ≡b 3 (mod 7), 其中b 1 = 1,b 2 = 0,3,b 3 = 2,6, 由孙子定理得原同余方程的解为x ≡13,55,58,100 (mod 105)。 2、判断同余方程x 2≡42(mod 107)是否有解? 11074217 271071107713231071107311072107 710731072107732107422110721721107213)(=∴-=-=-==-=-=-==??≡-?--?-)()()()(),()()()(),()())()(( )(解: 故同余方程x 2≡42(mod 107)有解。 3、求(127156+34)28除以111的最小非负余数。

初等数论第2版习题答案(可编辑修改word版)

3 b b b 3 b 第一章 §1 1 证明: a 1 , a 2 , a n 都是 m 的倍数。 ∴存在 n 个整数 p 1, p 2 , p n 使 a 1 = p 1m 1 , a 2 = p 2 m 2 , , a n = p n m n 又 q 1, q 2 , , q n 是任意 n 个整数 ∴ q 1a 1 + q 2 a 2 + + q n a n = ( p 1q 1 + q 2 p 2 + + q n p n )m 即 q 1a 1 + q 2 a 2 + + q n a n 是 m 的整数 2 证: n (n + 1)(2n + 1) = n (n + 1)(n + 2 + n - 1) = n (n + 1)(n + 2) + (n - 1)n (n + 1) 6 / n (n + 1)(n + 2),6 /(n - 1)n (n + 1) ∴ 6 / n (n + 1)(n + 2) + (n - 1)n (n + 1) 从而可知 6 / n (n + 1)(2n + 1) 3 证: a , b 不全为0 ∴在整数集合 S = {ax + by | x , y ∈ Z }中存在正整数,因而 有形如 ax + by 的最小整数 ax 0 + by 0 ?x , y ∈ Z ,由带余除法有 ax + by = (ax 0 + by 0 )q + r ,0 ≤ r < ax 0 + by 0 则 r = (x - x 0 q )a + ( y - y 0 q )b ∈ S ,由 ax 0 + by 0 是 S 中的最小整数知 r = 0 ∴ ax 0 + by 0 / ax + by ax 0 + by 0 / ax + by ∴ ax 0 + by 0 /(a , b ). ∴(a , b ) / ax 0 + by 0 下证 P 8 第二题 ( x , y 为任意整数) 又有(a , b ) / a ,(a , b ) / b 故 ax 0 + by 0 = (a , b ) ∴ ax 0 + by 0 / a , ax 0 + by 0 / b 4 证:作序列 ,- ,- b ,- ,0, , b , 2 2 2 2 , 则 a 必在此序列的某两项之间

初等数论试卷和答案

初等数论试卷和答案 Company Document number:WTUT-WT88Y-W8BBGB-BWYTT-19998

初等数论考试试卷1 一、单项选择题(每题3分,共18分) 1、如果a b ,b a ,则( ). A b a = B b a -= C b a ≤ D b a ±= 2、如果n 3,n 5,则15( )n . A 整除 B 不整除 C 等于 D 不一定 3、在整数中正素数的个数( ). A 有1个 B 有限多 C 无限多 D 不一定 4、如果)(mod m b a ≡,c 是任意整数,则 A )(mod m bc ac ≡ B b a = C ac T )(mod m bc D b a ≠ 5、如果( ),则不定方程c by ax =+有解. A c b a ),( B ),(b a c C c a D a b a ),( 6、整数5874192能被( )整除. A 3 B 3与9 C 9 D 3或9 二、填空题(每题3分,共18分) 1、素数写成两个平方数和的方法是( ). 2、同余式)(mod 0m b ax ≡+有解的充分必要条件是( ). 3、如果b a ,是两个正整数,则不大于a 而为b 的倍数的正整数的个数为 ( ). 4、如果p 是素数,a 是任意一个整数,则a 被p 整除或者( ). 5、b a ,的公倍数是它们最小公倍数的( ). 6、如果b a ,是两个正整数,则存在( )整数r q ,,使r bq a +=,b r ≤0.

三、计算题(每题8分,共32分) 1、求[136,221,391]= 2、求解不定方程144219=+y x . 3、解同余式)45(mod 01512≡+x . 4、求 ??? ??563429,其中563是素数. (8分) 四、证明题(第1小题10分,第2小题11分,第3小题11分,共 32分) 1、证明对于任意整数n ,数6233 2n n n ++是整数. 2、证明相邻两个整数的立方之差不能被5整除. 3、证明形如14-n 的整数不能写成两个平方数的和. 试卷1答案 一、单项选择题(每题3分,共18分) 1、D. 2、A 3、C 4、A 5、A 6、B 二、填空题(每题3分,共18分) 1、素数写成两个平方数和的方法是(唯一的). 2、同余式)(mod 0m b ax ≡+有解的充分必要条件是(b m a ),(). 3、如果b a ,是两个正整数,则不大于a 而为b 的倍数的正整数的个数为( ][b a ). 4、如果p 是素数,a 是任意一个整数,则a 被p 整除或者( 与p 互素 ). 5、b a ,的公倍数是它们最小公倍数的( 倍数 ). 6、如果b a ,是两个正整数,则存在( 唯一 )整数r q ,,使r bq a +=,b r ≤0.

2013年春_西南大学《初等数论》作业及答案(共4次_已整理)

2013年春西南大学《初等数论》作业及答案(共4次,已整理) 第一次作业 1、设n,m为整数,如果3整除n,3整除m,则9()mn。 A:整除 B:不整除 C:等于 D:小于 正确答案:A 得分:10 2、整数6的正约数的个数是()。 A:1 B:2 C:3 D:4 正确答案:D 得分:10 3、如果5|n ,7|n,则35()n 。 A:不整除 B:等于 C:不一定 D:整除 正确答案:D 得分:10 4、如果a|b,b|a ,则()。 A:a=b B:a=-b C:a=b或a=-b D:a,b的关系无法确定 正确答案:C 得分:10 5、360与200的最大公约数是()。 A:10 B:20 C:30 D:40 正确答案:D 得分:10 6、如果a|b,b|c,则()。 A:a=c B:a=-c C:a|c D:c|a

正确答案:C 得分:10 7、1到20之间的素数是()。 A:1,2,3,5,7,11,13,17,19 B:2,3,5,7,11,13,17,19 C:1,2,4,5,10,20 D:2,3,5,7,12,13,15,17 正确答案:B 得分:10 8、若a,b均为偶数,则a + b为()。 A:偶数 B:奇数 C:正整数 D:负整数 正确答案:A 得分:10 9、下面的()是模12的一个简化剩余系。 A:0,1,5,11 B:25,27,13,-1 C:1,5,7,11 D:1,-1,2,-2 正确答案:C 得分:10 10、下面的()是模4的一个完全剩余系。 A:9,17,-5,-1 B:25,27,13,-1 C:0,1,6,7 D:1,-1,2,-2 正确答案:C 得分:10 11、下面的()是不定方程3x + 7y = 20的一个整数解。 A:x=0,y=3 B:x=2,y=1 C:x=4,y=2 D:x=2,y=2 正确答案:D 得分:10 12、设a,b,c,d是模5的一个简化剩余系,则a+b+c+d对模5同余于()。 A:0 B:1 C:2 D:3 正确答案:A 得分:10 13、使3的n次方对模7同余于1的最小的正整数n等于()。 A:6 B:2

初等数论试题

2 010年7月高等教育自学考试 初等数论试题 课程代码:10021 一、单项选择题(本大题共5小题,每小题2分,共10分) 在每小题列出的四个备选项中只有一个是符合题目要求的,请将其代码填写在题后的括号内。错选、多选或未选均无分。 1.-30被-9除的余数是() A.-3 C.3 2.下列给出的数中是合数的是() A.1063 C.1093 1000 3.400 xx5的幂指数是() B.-6 D.6 B.1073 D.1103

A.1 C.3B.2 D.4 4.不能表示为5x+7y(x,y是非负整数)的最大整数是() A.23 C.25B.24 D.26 5.下列给出的素数模数中,3是平方非剩余的是() A.37 C.53 二、填空题(本大题共10小题,每小题3分,共30分) 请在每小题的空格中填上正确答案。错填、不填均无分。 1.60480的标准分解式为___. 2.μ (50400)=___. 3.π( 55.5)=___. 4.对任意的正整数n,最大公因数(12n+1,30n+3)=___. 5.若(n)=4,则n=___. 6.同余方程6x≡7(mod 23)的解是___. 7.不定方程6x+9y=30的通解是___.

8.写出模10的一个最小的非负简化剩余系,并要求每项都是7的倍数,则此简化剩余系为 B.47 D.59 ___. 9.326 被50除的余数是___. 10.xxM 23是___(填素数或合数). 三、计算题(本大题共4小题,每小题10分,共40分) 1.已知两正整数中,每一个除以它们的最大公约数所得的商之和等于18,它们的最小公倍数等于975,求这两个数。 2.有一队士兵,若三人一组,则余1人;若五人一组,则缺2人;若十一人一组,则余3人。 已知这队士兵不超过170人,问这队士兵有几人? 3.求正整数x,使x2-1216是完全平方数。 4.已知563是素数,判断不定方程x2+563y=429是否有整数解。 四、证明题(本大题共2小题,每小题10分,共20分) 1.证明当n为整数时,504|n9-n3。 2.设(a,m)=1,若x通过模m的完全剩余系,则ax+b也通过模m的完全剩余系.

初等数论 1 习题参考答案

附录1 习题参考答案 第一章习题一 1. (ⅰ) 由a b知b = aq,于是b = (a)(q),b = a(q)及b = (a)q,即a b,a b及a b。反之,由a b,a b及a b 也可得a b; (ⅱ) 由a b,b c知b = aq1,c = bq2,于是c = a(q1q2),即a c; (ⅲ) 由b a i知a i= bq i,于是a1x1a2x2a k x k = b(q1x1 q2x2q k x k),即b a1x1a2x2a k x k;(ⅳ) 由b a知a = bq,于是ac = bcq,即bc ac; (ⅴ) 由b a知a = bq,于是|a| = |b||q|,再由a 0得|q| 1,从而|a| |b|,后半结论由前半结论可得。 2. 由恒等式mq np= (mn pq) (m p)(n q)及条件m p mn pq可知m p mq np。 3. 在给定的连续39个自然数的前20个数中,存在两个自然数,它们的个位数字是0,其中必有一个的十位数字不是9,记这个数为a,它的数字和为s,则a, a 1, , a 9, a 19的数字和为s, s 1, , s 9, s 10,其中必有一个能被11整除。 4. 设不然,n1= n2n3,n2p,n3p,于是n = pn2n3p3,即p3n,矛盾。 5. 存在无穷多个正整数k,使得2k1是合数,对于这样的k,(k1)2

不能表示为a2p的形式,事实上,若(k 1)2= a2p,则(k 1 a)( k 1 a) = p,得k 1 a = 1,k 1 a = p,即p = 2k 1,此与p为素数矛盾。 第一章习题二 1. 验证当n =0,1,2,… ,11时,12|f(n)。 2.写a = 3q1r1,b = 3q2r2,r1, r2 = 0, 1或2,由3a2b2 = 3Q r12r22知r1 = r2 = 0,即3a且3b。 3.记n=10q+r, (r=0,1,…,9),则n k+4-n k被10除的余数和r k+4-r k=r k(r4-1)被10 除的余数相同。对r=0,1,…,9进行验证即可。 4. 对于任何整数n,m,等式n2 (n 1)2 = m2 2的左边被4除的余数为1,而右边被4除的余数为2或3,故它不可能成立。 5 因a4 3a2 9 = (a2 3a 3)( a2 3a 3),当a = 1,2时,a2 3a 3 = 1,a4 3a2 9 = a2 3a 3 = 7,13,a4 3a2 9是素数;当a 3时,a2 3a 3 > 1,a2 3a 3 > 1,a4 3a2 9是合数。 6. 设给定的n个整数为a1, a2, , a n,作 s1 = a1,s2 = a1a2,,s n = a1a2a n, 如果s i中有一个被n整除,则结论已真,否则存在s i,s j,i < j,使得s i与s j 被n除的余数相等,于是n s j s i = a i + 1a j。

最新初等数论试卷,最全面的答案,包括截图

初等数论考试试卷 一、 单项选择题:(1分/题×20题=20分) 1.设x 为实数,[]x 为x 的整数部分,则( A ) A.[][]1x x x ≤<+; B.[][]1x x x <≤+; C.[][]1x x x ≤≤+; D.[][]1x x x <<+. 2.下列命题中不正确的是( B ) A.整数12,,,n a a a 的公因数中最大的称为最大公因数; B.整数12,,,n a a a 的公倍数中最小的称为最小公倍数 【有最小的吗?】 C.整数a 与它的绝对值有相同的倍数 D.整数a 与它的绝对值有相同的约数 3.设二元一次不定方程ax by c +=(其中,,a b c 是整数,且,a b 不全为零)有一整数解 ()00,,,x y d a b =,则此方程的一切解可表为( C ) A.00,,0,1,2,;a b x x t y y t t d d =- =+ =±± B.00,,0,1,2,;a b x x t y y t t d d =+= -=±± C.00,,0,1,2,;b a x x t y y t t d d =+= -=±± D.00,,0,1,2,;b a x x t y y t t d d =-= -=±± 4.下列各组数中不构成勾股数的是( D ) A.5,12,13; B.7,24,25; C.3,4,5; D.8,16,17 5.下列推导中不正确的是( D ) A.()()()11221212mod ,mod mod ;a b m a b m a a b b m ≡≡?+≡+ B.()()()11221212mod ,mod mod ;a b m a b m a a bb m ≡≡?≡ C.()()111212mod mod ;a b m a a b a m ≡?≡ D.()()112211mod mod .a b m a b m ≡?≡ 6.模10的一个简化剩余系是( D )

初等数论练习题

初等数论练习题 信阳职业技术学院 2010年12月

初等数论练习题一 一、填空题 1、d(2420)=___________; ?(2420)=___________。 2、设a,n 是大于1的整数,若a n -1是质数,则a=___________。 3、模9的绝对最小完全剩余系是___________。 4、同余方程9x+12≡0(mod 37)的解是__________。 5、不定方程18x-23y=100的通解是___________。 6、分母是正整数m 的既约真分数的个数为_______。 7、18100被172除的余数是___________。 8、?? ? ??10365 =___________。 9、若p 是素数,则同余方程x p 1 1(mod p )的解数为 。 二、计算题 1、解同余方程:3x 2 11x 200 (mod 105)。 2、判断同余方程x 2≡42(mod 107)是否有解 3、求(127156+34)28除以111的最小非负余数。 三、证明题 1、已知p 是质数,(a,p )=1,证明: (1)当a 为奇数时,a p-1+(p-1)a ≡0 (mod p); (2)当a 为偶数时,a p-1-(p-1)a ≡0 (mod p)。 2、设a 为正奇数,n 为正整数,试证n 2a ≡1(mod 2n+2)。 3、设p 是一个素数,且1≤k ≤p-1。证明:k p 1C - (-1 )k (mod p )。 4、设p 是不等于3和7的奇质数,证明:p 6≡1(mod 84)。

初等数论练习题二 一、填空题 1、d(1000)=__________;σ(1000)=__________。 2、2010!的标准分解式中,质数11的次数是__________。 3、费尔马(Fermat)数是指Fn=n 22+1,这种数中最小的合数Fn 中的n=_________。 4、同余方程13x ≡5(mod 31)的解是__________。 5、分母不大于m 的既约真分数的个数为_________。 6、设7∣(80n -1),则最小的正整数n=__________。 7、使41x+15y=C 无非负整数解的最大正整数C=__________。 8、?? ? ??10146=__________。 9、若p 是质数,n p 1,则同余方程x n 1 (mod p ) 的解数为 。 二、计算题 1、试求2004 2003 2002被19除所得的余数。 2、解同余方程3x 144x 10 6x 180 (mod 5)。 3、已知a=5,m=21,求使a x 1 (mod m)成立的最小自然数x 。 三、证明题 1、试证13|(54m +46n +2000)。(提示:可取模13进行计算性证明)。 2、证明Wilson 定理的逆定理:若n > 1,并且(n 1)! 1 (mod n ),则n 是素数。 3、证明:设p s 表示全部由1组成的s 位十进制数,若p s 是素数,则s 也是一个素数。 4、证明:若2p 1是奇素数,则 (p !)2 ( 1)p 0 (mod 2p 1)。 5、设p 是大于5的质数,证明:p 4≡1(mod 240)。

初等数论第2版习题答案

第一章 §1 1 证明:n a a a ,,21 都是m 的倍数。 ∴存在n 个整数n p p p ,,21使 n n n m p a m p a m p a ===,,,222111 又n q q q ,,,21 是任意n 个整数 m p q p q q p a q a q a q n n n n )(22112211+++=+++∴ 即n n a q a q a q +++ 2211是m 的整数 2 证: )12)(1()12)(1(-+++=++n n n n n n n )1()1()2)(1(+-+++=n n n n n n )1()1/(6),2)(1(/6+-++n n n n n n )1()1()2)(1(/6+-+++∴n n n n n n 从而可知 )12)(1(/6++n n n 3 证: b a , 不全为0 ∴在整数集合{}Z y x by ax S ∈+=,|中存在正整数,因而 有形如by ax +的最小整数00by ax + Z y x ∈?,,由带余除法有00000,)(by ax r r q by ax by ax +<≤++=+ 则 S b q y y a q x x r ∈-+-=)()(00,由00by ax +是S 中的最小整数知0=r by ax by ax ++∴/00 下证8P 第二题 by ax by ax ++/00 (y x ,为任意整数) b by ax a by ax /,/0000++∴ ).,/(00b a by ax +∴ 又有b b a a b a /),(,/),( 00/),(by ax b a +∴ 故),(00b a by ax =+ 4 证:作序列 ,2 3, ,2 , 0,2 ,,2 3,b b b b b b - -- 则a 必在此序列的某两项之间

初等数论试卷

初等数论试卷 一、 单项选择题:(1分/题×20题=20分) 1.设x 为实数,[]x 为x 的整数部分,则( A ) A.[][]1x x x ≤<+; B.[][]1x x x <≤+; C.[][]1x x x ≤≤+; D.[][]1x x x <<+. 2.下列命题中不正确的是( B ) A.整数12,, ,n a a a 的公因数中最大的称为最大公因数; B.整数12,,,n a a a 的公倍数中最小的称为最小公倍数 C.整数a 与它的绝对值有相同的倍数 D.整数a 与它的绝对值有相同的约数 3.设二元一次不定方程ax by c +=(其中,,a b c 是整数,且,a b 不全为零)有一整数解()00,,,x y d a b =,则此方程的一切解可表为( C ) A.00,,0,1,2,;a b x x t y y t t d d =-=+=±± B.00,,0,1,2,;a b x x t y y t t d d =+=-=±± C.00,,0,1,2,;b a x x t y y t t d d =+=-=±± D.00,,0,1,2,;b a x x t y y t t d d =-=-=±± 4.下列各组数中不构成勾股数的是( D ) A.5,12,13; B.7,24,25; C.3,4,5; D.8,16,17 5.下列推导中不正确的是( D ) A.()()()11221212mod ,mod mod ;a b m a b m a a b b m ≡≡?+≡+ B.()()()11221212mod ,mod mod ;a b m a b m a a bb m ≡≡?≡ C.()()111212mod mod ;a b m a a b a m ≡?≡ D.()()112211mod mod .a b m a b m ≡?≡ 6.模10的一个简化剩余系是( D ) A.0,1,2,,9; B.1,2,3,,10;

初等数论习题集

《初等数论》习题集 第1章 第 1 节 1. 证明定理1。 2. 证明:若m - p ∣mn + pq ,则m - p ∣mq + np 。 3. 证明:任意给定的连续39个自然数,其中至少存在一个自然数,使得这个自然数的数字和能被11整除。 4. 设p 是n 的最小素约数,n = pn 1,n 1 > 1,证明:若p >3n ,则n 1是素数。 5. 证明:存在无穷多个自然数n ,使得n 不能表示为 a 2 + p (a > 0是整数,p 为素数) 的形式。 第 2 节 1. 证明:12∣n 4 + 2n 3 + 11n 2 + 10n ,n ∈Z 。 2. 设3∣a 2 + b 2,证明:3∣a 且3∣b 。 3. 设n ,k 是正整数,证明:n k 与n k + 4的个位数字相同。 4. 证明:对于任何整数n ,m ,等式n 2 + (n + 1)2 = m 2 + 2不可能成立。 5. 设a 是自然数,问a 4 - 3a 2 + 9是素数还是合数? 6. 证明:对于任意给定的n 个整数,必可以从中找出若干个作和,使得这个和能被n 整除。 第 3 节 1. 证明定理1中的结论(ⅰ)—(ⅳ)。 2. 证明定理2的推论1, 推论2和推论3。 3. 证明定理4的推论1和推论3。 4. 设x ,y ∈Z ,17∣2x + 3y ,证明:17∣9x + 5y 。 5. 设a ,b ,c ∈N ,c 无平方因子,a 2∣b 2c ,证明:a ∣b 。 6. 设n 是正整数,求1223212C ,,C ,C -n n n n 的最大公约数。 第 4 节 1. 证明定理1。 2. 证明定理3的推论。 3. 设a ,b 是正整数,证明:(a + b )[a , b ] = a [b , a + b ]。 4. 求正整数a ,b ,使得a + b = 120,(a , b ) = 24,[a , b ] = 144。 5. 设a ,b ,c 是正整数,证明: ) ,)(,)(,(),,(],][,][,[],,[2 2a c c b b a c b a a c c b b a c b a =。 6. 设k 是正奇数,证明:1 + 2 + + 9∣1k + 2k + + 9k 。 第 5 节 1. 说明例1证明中所用到的四个事实的依据。 2. 用辗转相除法求整数x ,y ,使得1387x - 162y = (1387, 162)。

初等数论试卷模拟试题和答案

初等数论试卷一 一、 单项选择题:(1分/题×20题=20分) 1.设x 为实数,[]x 为x 的整数部分,则( ) A.[][]1x x x ≤<+; B.[][]1x x x <≤+; C.[][]1x x x ≤≤+; D.[][]1x x x <<+. 2.下列命题中不正确的是( ) A.整数12,,,n a a a 的公因数中最大的称为最大公因数; B.整数12,, ,n a a a 的公倍数中最小的称为最小公倍数 C.整数a 与它的绝对值有相同的倍数 D.整数a 与它的绝对值有相同的约数 3.设二元一次不定方程ax by c +=(其中,,a b c 是整数,且,a b 不全为零)有一整数解 ()00,,,x y d a b =,则此方程的一切解可表为( ) A.00,,0,1,2,;a b x x t y y t t d d =- =+ =±± B.00,,0,1,2, ;a b x x t y y t t d d =+= -=±± C.00,,0,1,2, ;b a x x t y y t t d d =+= -=±± D.00,,0,1,2, ;b a x x t y y t t d d =-= -=±± 4.下列各组数中不构成勾股数的是( ) A.5,12,13; B.7,24,25; C.3,4,5; D.8,16,17 5.下列推导中不正确的是( ) A.()()()11221212mod ,mod mod ;a b m a b m a a b b m ≡≡?+≡+ B.()()()11221212mod ,mod mod ;a b m a b m a a bb m ≡≡?≡ C.()()111212mod mod ;a b m a a b a m ≡?≡ D.()()112 2 11mod mod .a b m a b m ≡?≡ 6.模10的一个简化剩余系是( ) A.0,1,2, ,9; B.1,2,3,,10;

初等数论习题与答案、及测试卷

1 证明:n a a a ,,21 都是m 的倍数。 ∴存在n 个整数n p p p ,,21使 n n n m p a m p a m p a ===,,,222111 又n q q q ,,,21 是任意n 个整数 m p q p q q p a q a q a q n n n n )(22112211+++=+++∴ 即n n a q a q a q +++ 2211是m 的整数 2 证: )12)(1()12)(1(-+++=++n n n n n n n )1()1()2)(1(+-+++=n n n n n n )1()1/(6),2)(1(/6+-++n n n n n n 1()1()2)(1(/6+-+++∴n n n n n n 从而可知 12)(1(/6++n n n 3 证: b a , 不全为0 ∴在整数集合{}Z y x by ax S ∈+=,|中存在正整数,因而 有形如by ax +的最小整数00by ax + Z y x ∈?,,由带余除法有00000,)(by ax r r q by ax by ax +<≤++=+ 则b q y y a q x x r ∈-+-=)()(00,由00by ax +是S 中的最小整数知0=r ax by ax + +∴/00 下证8P 第二题 by ax by ax ++/00 (y x ,为任意整数) b by ax a by ax /,/0000++∴ ,/(00b a by ax +∴ 又有b b a a b a /),(,/),( 0/),(by ax b a +∴ 故),(00b a by ax =+ 4 证:作序列 ,2 3, ,2 , 0,2 ,,2 3,b b b b b b - -- 则a 必在此序列的某两项之间

(完整word版)初等数论练习题一(含答案)

《初等数论》期末练习二 一、单项选择题 1、=),0(b ( ). A b B b - C b D 0 2、如果1),(=b a ,则),(b a ab +=( ). A a B b C 1 D b a + 3、小于30的素数的个数( ). A 10 B 9 C 8 D 7 4、如果)(mod m b a ≡,c 是任意整数,则 A )(mod m bc ac ≡ B b a = C (mod )ac bc m ≡/ D b a ≠ 5、不定方程210231525=+y x ( ). A 有解 B 无解 C 有正数解 D 有负数解 6、整数5874192能被( )整除. A 3 B 3与9 C 9 D 3或9 7、如果a b ,b a ,则( ). A b a = B b a -= C b a ≥ D b a ±= 8、公因数是最大公因数的( ). A 因数 B 倍数 C 相等 D 不确定 9、大于20且小于40的素数有( ). A 4个 B 5个 C 2个 D 3个 10、模7的最小非负完全剩余系是( ). A -3,-2,-1,0,1,2,3 B -6,-5,-4,-3,-2,-1 C 1,2,3,4,5,6 D 0,1,2,3,4,5,6 11、因为( ),所以不定方程71512=+y x 没有解. A [12,15]不整除7 B (12,15)不整除7 C 7不整除(12,15) D 7不整除[12,15] 12、同余式)593(mod 4382≡x ( ). A 有解 B 无解 C 无法确定 D 有无限个解 二、填空题 1、有理数 b a ,0,(,)1a b a b <<=,能写成循环小数的条件是( ). 2、同余式)45(mod 01512≡+x 有解,而且解的个数为( ). 3、不大于545而为13的倍数的正整数的个数为( ). 4、设n 是一正整数,Euler 函数)(n ?表示所有( )n ,而且与n ( )的正整数的个数. 5、设b a ,整数,则),(b a ( )=ab . 6、一个整数能被3整除的充分必要条件是它的( )数码的和能被3整除. 7、+=][x x ( ). 8、同余式)321(mod 75111≡x 有解,而且解的个数( ). 9、在176与545之间有( )是17的倍数.

初等数论试卷

一、判断题(对的写A ,错的写B ,3'1030?=) 1.12,,,k a a a 两两互素可以推出12,,,k a a a 互素,反之亦真。 ( ) 2.设10n n N a a a -=是整数N 的十进制表示,则0 1111(1)n i i i N a =?-∑。 ( ) 3.设,,a b m 是整数,(,)1a m =,若x 通过模m 的简化剩余系,则ax b +也通过模m 的简化剩余系。 ( ) 4.对于正整数k ,Euler 函数()k ?的值等于模k 简化剩余系中元素的个数。 ( ) 5.形如65n +的素数有无穷多个。 ( ) 6.32514805112133=????是51480的标准分解式。 ( ) 7. 已知(,,)x y z 是不定方程222x y z +=满足(,)1x y =的正整数解,则,x y 有不同的奇偶性。 ( ) 8.同余方程322310(mod5)x x x -+-≡的解数小于3。 ( ) 9. 3,5,9(mod14)x ≡是模14的全部原根。 ( ) 10.设,x y 是任意实数,则[][][]x y x y +=+。 ( ) 二、填空(3'1030?=) 1.159313被7除的余数是 。 2.使12347!被35k 整除的最大的k = 。 3.用(,)a b ,[,]a b 分别表示整数,a b 的最大公约数和最小公倍数,则[,](,)a b a b = 。 4.设n 是正整数,12,,,k p p p 是它的全部素因数,则 ()n ?= 。 5.同余方程2 1(mod61)x ≡-的解数是 。 6.设,a b 是整数,0(mod )a m ≠,则同余方程(mod )ax b m ≡有解的充要条件是 。若有解,则恰有 个解,mod m 。 7.模11的所有二次剩余是 。

初等数论计算题答案

初等数论第三次作业 计算题 1. 求75与105的最大公因数。 解:因为75 = 3错误!未找到引用源。52,105 = 3错误!未找到引用源。5错误!未找到引用源。7, 所以75与105的最大公因数是15。 2. 求66与121的最大公因数。 解:因为66=6×11,121=11×11, 所以66与121的最大公因数是11 3.求不定方程3x - 4y = 1的一切整数解。 答;因为(3,4)= 1,所以不定方程有整数解。 观察知x = 3,y = 2是其一个整数解。 由公式知其一切整数解为???+=+=t y t x 3243,t 为整数。 4.求不定方程7x + 2y = 1的一切整数解。 答;因为(7,2)=1,1|1,所以不定方程有解。观察知其一个整数解是 0013 x y =??=-?。 于是其一切整数解为1237x t y t =+??=--? ,t 取一切整数。 5.解同余式3x ≡ 1 (mod 7)。 答;因为(3,7)= 1,所以同余式有解且有一个解。 由3x - 7y = 1得???+=+=t y t x 3275, 所以同余式的解为)7(mod 5≡x 6.解同余式3x ≡ 8 (mod 10)。

答;因为(3,10)=1,1|8,所以同余式有解,并且只有一个解。由3108x y -=得 一个解00 61x y =??=?,所以同余式的解为6(mod10)x ≡。 7.解同余式28x ≡ 21 (mod 35)。 答:因为(28,35) = 7,而7|21,所以同余式28x ≡ 21(mod 35)有解,且有7个解。同余式28x ≡ 21(mod 35)等价于4x ≡ 3(mod 5),解4x ≡ 3(mod 5)得x ≡ 2(mod 5),故同余式28x ≡ 21(mod 35)的7个解为x ≡ 2,7,12,17,22,27,32(mod 35)。 8.解同余式组: ???≡≡) 5(mod 2)3(mod 1x x 。 答;由)3(mod 1≡x 得13+=k x ,将其代入)5(mod 2≡x 得)5(mod 213≡+k , 解得)5(mod 2≡k ,即25+=t k , 所以715+=t x ,所以解为)15(mod 7≡x 。 9. 求不定方程3x + 2y = 2的一切整数解。 解:因为(3,2) = 1,所以不定方程有整数解。 显然1,0==y x 是其一个特解, 所以不定方程的一切整数解为错误!未找到引用源。,其中t 取一切整数。 10.解同余式)5(mod 14≡x 答;因为(4,5)= 1,所以同余式有解且有一个解。 由4x - 5y = 1得???+=+=t y t x 3275, 所以同余式的解为)7(mod 5≡x

初等数论复习题题库及答案

《初等数论》本科 一 填空题(每空2分) 1.写出30以内的所有素数 2,3,5,7,11,13,17,19,23,29 . 2.,( ,)(,)(,) a b a b a b a b =设是任意两个不为零的整数,则 1 . 3.若,a b 是非零整数,则a 与b 互素的充要条件是存在整数,x y ,适1ax by += 4.写出180的标准分解式是 22235?? ,其正约数个数有 (2+1)(2+1)(1+1)=18个. 5.,1,2,,a b a b L 设与是正整数则在中能被整除的整数恰有 []a b 个. 6.设,a b 是非零整数,c 是整数,方程ax by c +=有整数解(,x y )的充要条件是 (,)|a b c 7. 若整数集合A 是模m 的完全剩余系,则A 中含有 m 个整数. 8.?(3)= 2 ;?(4)= 2 . 9.当p 素数时,(1)()p ?= 1p - ;(2)()k p ?= 1k k p p -- . 10.(),(,)1,1m m a m a ?=-≡设是正整数则 0 (mod ).m 11.,,p p a a a -≡设是素数则对于任意的整数有 0 (mod ).p 12.已知235(mod7)x +≡,则x ≡ 1 (mod7). 13.同余方程22(mod 7)x ≡的解是 4(mod7) . 14.同余方程2310120(mod 9)x x ++≡的解是 .X=6. . 15.(,)1n p =若,n p 是模的二次剩余的充要条件是 -121(mod ).p n p ≡ . 16.(,)1n p =若,n p 是模的二次非剩余的充要条件是 -12 1(mod ).p n p ≡- . 17.3()=5 -1 ; 4 ()=5 1 . 18.,p 设是奇素数则2 ()p = 218(1).p -- . 19.,p 设是奇素数则1()p = 1 ;-1 ()p = -1 2(-1).p . 20. 5()=9 1 ; 2 ()=45 -1 . 二 判断题(判断下列结论是否成立,每题2分). 1. ||,|a b a c x y Z a bx cy ?∈+且对任意的有.成立 2. (,)(,),[,][,]a b a c a b a c ==若则.不成立

初等数论试卷

一、填空题(本大题共10小题,每小题4分,共40分)请在每小题的空格中填上正确答案。错填、不填均无分。 1.μ(2002)=_________; d(2002)=_________. 2.自然数225,226,…,240中的素数是_________. 3.n+2,2n+3,3n+1中必定互素的一组数是_________. 4.模7的绝对值最小简化剩余系是_________. 5.同余方程16x ≡6(mod 46)的解是_________. 6.不定方程3x+4y=5的通解是_________. 7.17|(2002n -1),则正整数n 的最小值是_________. 8.满足?(n) =20的n 有多个,其中两个是_________. 9.弗罗贝纽斯(Frobenius)问题可表述为_________. 10.?? ? ??17954 =_________. 二、计算题(本大题共3小题,第1,2小题各7分,第3小题9分,共23分) 1.判断下面同余方程组是否有解,如有解则求出其解: ?? ???≡≡≡9).5(mod x 20),7(mod x 15),2(mod x 2.试求不定方程y 2+x=x 2 +y-22的所有正整数解. 3.判断同余方程x 2≡62(mod 113)是否有解,如有解,则使用高斯(Gauss)逐步淘汰法求其解. 三、论证题(本大题共4小题,第1,2小题各8分,第3小题10分,第4题11分,共37 分) 1.试证一个正整数的平方,必与该正整数的各位数码字的和的平方,关于模9同余。 2.设(a,m)=1,x 通过模m 的一个简化剩余系,试证ax 也通过模m 的简化剩余系. 3.设F n =n 22+1,试证(F n ,F n+1)=1. 4.试证在两继自然数的平方之间,不存在四个自然数a

初等数论习题

https://www.360docs.net/doc/fb1712026.html, 《初等数论》习题集 第1章 第 1 节 1. 证明定理1。 2. 证明:若m - p ∣mn + pq ,则m - p ∣mq + np 。 3. 证明:任意给定的连续39个自然数,其中至少存在一个自然数,使得这个自然数的数字和能被11整除。 4. 设p 是n 的最小素约数,n = pn 1,n 1 > 1,证明:若p >3n ,则n 1是素数。 5. 证明:存在无穷多个自然数n ,使得n 不能表示为 a 2 + p (a > 0是整数,p 为素数) 的形式。 第 2 节 1. 证明:12∣n 4 + 2n 3 + 11n 2 + 10n ,n ∈Z 。 2. 设3∣a 2 + b 2,证明:3∣a 且3∣b 。 3. 设n ,k 是正整数,证明:n k 与n k + 4的个位数字相同。 4. 证明:对于任何整数n ,m ,等式n 2 + (n + 1)2 = m 2 + 2不可能成立。 5. 设a 是自然数,问a 4 - 3a 2 + 9是素数还是合数? 6. 证明:对于任意给定的n 个整数,必可以从中找出若干个作和,使得这个和能被n 整除。 第 3 节 1. 证明定理1中的结论(ⅰ)—(ⅳ)。 2. 证明定理2的推论1, 推论2和推论3。 3. 证明定理4的推论1和推论3。 4. 设x ,y ∈Z ,17∣2x + 3y ,证明:17∣9x + 5y 。 5. 设a ,b ,c ∈N ,c 无平方因子,a 2∣b 2c ,证明:a ∣b 。 6. 设n 是正整数,求1 223212C ,,C ,C -n n n n 的最大公约数。 第 4 节 1. 证明定理1。 2. 证明定理3的推论。 3. 设a ,b 是正整数,证明:(a + b )[a , b ] = a [b , a + b ]。 4. 求正整数a ,b ,使得a + b = 120,(a , b ) = 24,[a , b ] = 144。 5. 设a ,b ,c 是正整数,证明: ) ,)(,)(,(),,(],][,][,[],,[2 2a c c b b a c b a a c c b b a c b a =。 6. 设k 是正奇数,证明:1 + 2 + + 9∣1k + 2k + + 9k 。

相关文档
最新文档